Welcome to AZ for Spring Conference!
The Fast Track Spring 2014 - Issue 10
AN EMERGENCY MEDICINE PUBLICATION
Spring Spring Spring2014 2014 2014
The The Fast Fast Track Track
POLITICS
How will Washington D.C. affect your career?
EM-Trained Congressman Congressman EM-trained Speaks Out! Speaks
Perspectives: DO Day D.O. Dayon on the the Hill Hill Editorial: Needed in in Editorial: More More Law law needed Medical school School curricula Curricula medical
Fluid Management - Get the Breakdown Inside
page page 1 3 1
The Track TheFast Fast Track Editors & Publishers Spring 2014
Tanner Gronowski Drew Kalnow Joe Sorber Andrew Little Jeremy Lacocque Giles Gifford Danielle Turrin Kaitlin Fries Erin Sernoffsky
Issue Contributors Giles Gifford Steve Brandon Kaitlin Fries Jeremy Lacocque Brent Arnold Chase Ungs Danielle Turrin Joe Heck Greg Christiansen Nicholas Pettit Robert Culley Andrew Little Brian Lehnhof John Casey Esther Hwang Timothy Marcoux Jr Drew Kalnow
Interested in contributing? Let us know: FastTrack@ACOEP.org
T
Letter from the Editor
his year has certainly seen some new and exciting changes in the osteopathic profession. Through great effort, an agreement between the ACGME and the AACOM to have a single accreditation system for graduate medical education programs will be implemented beginning in July 2015. We are all curious to see just how these changes will affect us and our colleagues. It is important for us to acknowledge that this step forward could not have happened without the continued efforts of physicians and policy makers in navigating the political waters to find a solution in the best interests of our profession. It is with great pride that we present this edition to our readers. In this issue our contributors have delivered their standard quality case discussions complete with thorough learning points, but they were also encouraged to focus on political aspects of medicine. In response, we received many insightful submissions from a wide variety of authors. The selections include thoughts and experiences of students and residents from the recent DO Day On Capitol Hill, a viewpoint from inside the AOA Bureau on Federal Health Programs, a call to action from a highly respected attending physician and a special message from a State Congressman focusing on ways we can all positively impact the future of osteopathic medicine. On behalf of The Fast Track team, we are not only very grateful to our contributing authors but also to you, our readers. We sincerely hope that you not only learn from our publication, but that it motivates you to become more involved in the future of the profession. As Dante Alighieri once said, “The secret of getting things done is to act.� This remains just as true today as it did 700 years ago. Giles Gifford, OMS-II Pacific Northwest University of Health Sciences
CONTENTS Presidential Messages .................................... 04 By Kaitlin Fries and Steve Brandon
Printing of this issue sponsored by:
Rosh Review........................................................ 06 Emergency Medicine Board Style Questions
page 2 photo courtesy of Andrew Little Cover
Case Study: Encephalopathy ..................... 08 By Jeremy Lacocque
Spring 2014
The Fast Track
10
16
MANAGING STRESS by Brent Arnold
A CAUTIONARY TALE By Greg Christiansen
Tricks of the Trade ........................................... 11 By Chase Ungs
Pimpology ............................................................ 12 By Danielle Turrin
11
TRICKS OF THE TRADE by Chase Ungs
Fluid Management ........................................... 20 By Robert Culley
View from the Bottom ................................... 24 By Brian Lehnhof
Being Politically active ................................... 26 By John Casey
DO Day on The Hill ......................................... 28
36
CASE: ANGIOEDEMA by Drew Kalnow
40
RESIDENCY SPOTLIGHT page Midwestern University -
Managing Suspected Appendicitis .......... 30 By Esther Hwang
From the Journal of................................... 34 By Timothy Marcoux Jr.
Circling The Wagons ...................................... 39 By Andrew Little
Rosh Review Answers .................................... 42
Chicago, Illinois
3
The Fast Track
PRESIDENTIAL MESSAGE – Spring 2014
Resident Chapter
I
would like to start by congratulating all of the fourth-year medical students who successfully matched into their residency programs just a few short weeks ago. This is a huge accomplishment and you should all be proud of yourselves. Emergency medicine is the third largest specialty amongst AOA residencies, with 52 programs and 270 intern spots this year. This year only three of those spots went unmatched, and they were quickly filled post-match. We select from the cream of the crop, and whether you matched into your top choice or your last, you have done what many other talented individuals could not. When I think of this, I can’t help but feel grateful for the spot I have in this amazing specialty. The day after the match this year, I happened to have a shift with a fourthyear medical student who had not matched into an emergency medicine spot like he had hoped. His disappointment was palpable. Yet, he did not fail to match because of poor board scores or personality. In fact he is very intelligent and capable, and would have been a strong addition to any emergency program. The reality is that this specialty has become a very competitive one. In fact, I sometimes wonder how successful I would be now if I had to go through the match with the amazing candidates our programs interview. Every emergency medicine resident has worked hard and deserves the spot they now fill. Nevertheless, I encourage you all to recognize how lucky we are. Emergency medicine is a great specialty, and I can’t think of anything else I’d rather be doing. Sincerely, Steve Brandon, DO St. Mary Mercy Hospital
page 4
PRESIDENTIAL MESSAGE –
Student Chapter
Spring 2014
The Fast Track
W
elcome to another edition of the Fast Track! On behalf of the ACOEPSC, I would like to congratulate all of our fourth-year members who recently matched into emergency medicine. It must be a great feeling to know all your hard work has finally paid off. The end is in sight; in just a few short weeks you will walk across that stage and receive your diploma. We hope you enjoyed your time as a student member and will continue to be a part of our ACOEP family at the resident level. We wish you the best of luck on the next journey in your medical career. For the rest of us, we still have a lot of work to do. Each year obtaining an emergency medicine residency position grows increasingly more competitive and this year’s match data proved no different. While grades and board scores are important, we are hearing a lot more programs stress the importance of a well rounded CV and dedication to the field of Emergency Medicine. What better way to show your dedication than to become involved on a national level. Your student chapter officers have put together an agenda packed with ways to help strengthen your application and also provide you access to networking opportunities with program directors and residents from across the country. For those of you that are unable to attend, we will be posting podcasts from all the lectures and panel discussions to our website. Don’t forget another easy way to get involved is publishing an article here, in The Fast Track. As President of the ACOEP-SC one of my main goals is to help our members be highly competitive residency applicants. If you have any suggestions on ways we could better serve our members please don’t hesitate to contact any of the national officers including myself. We hope to see you in Scottsdale! Sincerely, Kaitlin Fries, OMS-III OU-HCOM
page 5
The Fast Track
Spring 2014
Emergency Medicine Review with 1. A 27-year-old woman presents with visual changes. She notes progressive worsening vision in her right eye over the last two days. She reports pain with movement of her eye. On examination, she has an afferent pupillary defect in that eye and a swollen optic disk on fundoscopy. What is the most likely diagnosis? A. Macular degeneration B. Optic neuritis C. Papilledema D. Retinal detachment
Find more questions like these by visiting roshreview.com
2. A 65-year-old, undomiciled man complains of foot numbness. He has been outside over the last several days during the rain, unable to change his shoes or socks. When his wet socks are removed, his feet are cold, cyanotic and edematous. What is his diagnosis?
page 6
A. Chilblain B. Frostbite C. Frostnip D. Immersion foot 3. A 57-year-old physicist presents after a radiation exposure. He is unsure of the amount of radiation. Decontamination already occurred and the patient is asymptomatic. At 48 hours, which of the following is best used for prognosis? A. Absolute lymphocyte count B. CT scan of the brain C. Geiger counter D. Platelet count 4. A 22-year-old woman with a history of depression and previous overdose presents with altered mental status. The patient is disoriented with mumbling speech. Her vitals are: T 101.2째F, HR 122, BP 130/82; RR 16. Physical examination is notable for dilated pupils, dry mucous membranes and flushed skin. On ECG, her QRS interval is 60 ms. Which ingested medication is most likely? A. Diphenhydramine B. Phenylephrine C. Salicylate D. Sertraline Find your Rosh Review Answers on page 42
2014
ACOEP SPRING
STUDENT CHAPTER CONFERENCE
SCOTTSDALE, ARIZONA APRIL 23 - 25
HIGHLIGHTS •LEADERSHIP ACADEMY •ATTENDING LECTURES •SKILLS LABS •RESIDENT AND PD PANELS •EVENING EVENTS WITH RESIDENTS
Spring 2014
The Fast Track
FOR MORE INFORMATION www.facebook.com/acoepsc TO REGISTER www.acoep.org page 7
The Fast Track
Spring 2014
Case Study:
Va l p roa te-I n d u ced Encephalopathy by Jeremy A. Lacocque, OMS-III @ CCOM
History of present illness: John is a 33-year-old male who presents to a Wisconsin community hospital emergency department at 0730 with dizziness, confusion and general weakness since 0500. John has anxiety, attention deficit hyperactivity disorder and major depressive disorder and takes methylphenidate, risperidone and valproic acid. John reported fatigue and confusion after an overnight shift in a factory. He said he often feels this way when exposed to fumes from the machines he works around, but after moving out of the area and noticing others did not feel the same way, he decided to come to the emergency department for further evaluation. He also reported difficulty walking to his boss’ car and to the ED because he felt so weak. In addition to his weakness, he reported feeling confused and more “out of it” than usual. He said he had trouble forming sentences at times, although he denies having trouble currently. He denies any similar previous episodes or co-workers with similar symptoms. He did say his son had nausea, vomiting and diarrhea, which resolved yesterday. He denies feeling ill, vomiting, diarrhea, constipation, problems voiding, pain, headache, changes in vision or sensation, although he did report feeling nauseous. He is allergic to “blood thinners” and wellbutrin. His medications include 20 mg Metadate qD, 2 mg risperidone at bedtime and 500mg depakote, five times per day for his bipolar disorder, ADHD and depression. He said after years of adjusting medications and not being fully compliant, he finally found a medication regimen he feels works, and for the past two years, he has been compliant and feels his normal self. Outside of John’s psychiatric disorders, John also has a past medical history of hemophilia factor II deficiency, sleep apnea and interstitial lung disease. His past surgical history includes two, left knee arthroscopies. John has smoked a pack per day of cigarettes since he was 13, giving him 20 pack years. He quit drinking one year ago, and “has not had even one drink” since then. He denies illicit drug use. He is recently divorced and lives in an apartment with his son. He works full time, overnight at a factory. Physical exam John’s blood pressure was 148/100, pulse 90, respiratory rate 18 with 98% SpO2 on room air. John was A&O x 3 and had a benign neuro exam. His HEENT, heart, lungs and abdominal exam were all unremarkable. Labs and imaging His head CT was unremarkable. His CBC revealed mild macrocytic anemia. Urinalysis, tox screen and a complete metabolic panel were unremarkable and revealed no transaminitis. His ammonia, however, was elevated at 62.5 (normal 11-35 μMol/L) and protime was slightly elevated at 11.3. INR and aPTT were normal. His salicylate and acetaminophen levels were undetectable. His valproic acid level was high normal at 97.9 (normal at this facility is 50-100 μg/mL).
page 8
Discussion Altered mental status has an extensive differential diagnosis as all emergency department physicians, residents and students are well aware of. Most can be categorized as secondary to toxic/metabolic, structural, infectious, abnormal vital signs or psychogenic causes. Since John denied any trauma, had no signs or symptoms of trauma, and had normal vital signs, he was worked up for toxic/metabolic encephalopathy, with psychogenic AMS being at the bottom of the list. While his valproic acid level was still in the “normal range,” it was higher than his past levels, and he was clearly encephalopathic.
Valproic acid (Depakote) is used to treat partial and generalized seizures and acute mania, and as prophylaxis for bipolar disorder and migraine headaches. Side effects include nausea (31% of patients), headache (31%), somnolence (16-20%), ataxia (<8%), confusion (<5%) and finally, hepatic toxicity and hyperammonemia, whose frequency is not defined, according to Medscape.1
Spring 2014
The Fast Track
Since John was on such a tight regimen of this medication and he had a history of being compliant, acute overdose of valproic acid did not seem likely. We believe John’s liver was impaired because of the accumulation of the toxic metabolites (propionic acid and 4-EN-VPA) often seen with chronic use of valproic acid, leading to hyperammonemia and hepatic encephalopathy. These toxic metabolites work by inhibiting mitochondrial carbamoyl phosphate synthetase, an enzyme necessary for ammonia elimination via the urea cycle.3 When this enzyme is impaired, ammonia levels accumulate, frequently resulting in encephalopathy.9 In addition, valproic acid may cause a rise in serum ammonia levels through interaction with carnitine, a cofactor necessary for mitochondrial long-chain fatty acid metabolism.10,12,13 Oral carnitine administration reverses carnitine deficiency, leads to resolution of hyperammonaemia, and improves lethargy in patients treated chronically with valproic acid.12-14 It also may hasten the resolution of coma, prevent hepatic dysfunction, and reverse metabolic abnormalities in patients with acute valproic acid intoxication.7,15 Proper carnitine therapy has shown no adverse effects.16 With this in mind, John seemed like a good candidate for a carnitine supplement. Other treatment options include naloxone in opioid-naive patients and activated charcoal in cases of acute overdose of valproic acid, although these have not been studied or proven to be as efficacious as carnitine therapy and suspending of valproic acid therapy. Assessment John is a 33-year-old male, who despite being closely adherent to a well-tuned and well-supervised medication regimen and a grossly benign workup and exam, had confusion and general weakness. Since his valproic acid level is high normal and he has hyperammonemia, a working diagnosis of valproic-acid related hepatic encephalopathy is made. Based on the above research, we decided to: Plan • Decrease depakote dosing to 250mg q5h for two days, then resume regular dosing of 500mg q5h assuming improvement in condition. • Levocarnitine supplementation, 2g, q8h until resolution of symptoms and cleared by psychiatrist with normalization of ammonia and valproic acid. • Follow up with psychiatry. Unfortunately, John was uninsured and was burdened with a $120 bill for the 30-day prescription of levocarnitine from the hospital’s pharmacy. While expensive, the carnitine supplement and reduction of Depakote dosing lead to the resolution of his symptoms and he and his other healthcare providers are now aware of his valproic-acid-induced encephalopathy and how to treat it. References: 1. Valproic acid. Medscape. 2013. Available at http://reference.medscape.com/drug/depakene-stavzor-valproic-acid-343024#4. Access February 27, 2014. 2. Gram L, Bentsen KD. Valproate: an updated review. Acta Neurol Scand 1985; 72:129. 3. Dupuis RE, Lichtman SN, Pollack GM. Acute valproic acid overdose. Clinical course and pharmacokinetic disposition of valproic acid and metabolites. Drug Saf 1990; 5:65. 4. Coulter DL, Allen RJ. Secondary hyperammonaemia: a possible mechanism for valproate encephalopathy. Lancet 1980; 1:1310. 5. Khoo SH, Leyland MJ. Cerebral edema following acute sodium valproate overdose. J Toxicol Clin Toxicol 1992; 30:209. 6. Anderson, GO, Ritland, S. Life threatening intoxication with sodium valproate. Clin Toxicol 1995; 33:279. 7. Ishikura H, Matsuo N, Matsubara M, et al. Valproic acid overdose and L-carnitine therapy. J Anal Toxicol 1996; 20:55. 8. Bryant AE 3rd, Dreifuss FE. Valproic acid hepatic fatalities. III. U.S. experience since 1986. Neurology 1996; 46:465. 9. Coulter DL, Allen RJ. Hyperammonemia with valproic acid therapy. J Pediatr 1981; 99:317. 10. Ishikura H, Matsuo N, Matsubara M, et al. Valproic acid overdose and L-carnitine therapy. J Anal Toxicol 1996; 20:55. 11.Bryant AE 3rd, Dreifuss FE. Valproic acid hepatic fatalities. III. U.S. experience since 1986. Neurology 1996; 46:465. 12. Ohtani Y, Endo F, Matsuda I. Carnitine deficiency and hyperammonemia associated with valproic acid therapy. J Pediatr 1982; 101:782. 13. Gidal BE, Inglese CM, Meyer JF, et al. Diet- and valproate-induced transient hyperammonemia: effect of L-carnitine. Pediatr Neurol 1997; 16:301. 14. Raby WN. Carnitine for valproic acid-induced hyperammonemia. Am J Psychiatry 1997; 154:1168. 15. Murakami K, Sugimoto T, Woo M, et al. Effect of L-carnitine supplementation on acute valproate intoxication. Epilepsia 1996; 37:687. 16. Mock CM, Schwetschenau KH. Levocarnitine for valproic-acid-induced hyperammonemic encephalopathy. Am J Health Syst Pharm 2012; 69:35. 17. Rivers, C. Valproic acid poisoning. UpToDate. 2014. Available at: http://www.uptodate.com/contents/valproic-acid-poisoning. Accessed February 23, 2014.
page 9
The Fast Track
Spring 2014
Stress–Can it be Healthy? by Brent Arnold, OMS-III @ William Carey COM
O
steopathic medicine teaches us to seek total health in our patients, encompassing mind, body, and spirit. To give optimal care, we need be at our best and incorporate these same tenets in ourselves. This is not achieved easily. We have chosen an exceptionally rewarding profession, yet the physical demands and utilization of our limited time in a 24-hour day can create significant stress. For most of us, it started well before medical school. If you think back to college, studying for class, preparing for the MCAT, filling out applications for medical school, and preparing for interviews; you remember. Then came what was supposed to be the hardest time in our lives, the first year of medical school, i.e. drinking water from the proverbial water hose. We made it through; however, it didn’t get any easier from there. The onslaught continued with second year, studying for boards during second year, starting clinical rotations, more boards, applying for residency, and an ever-increasing financial debt and professional responsibility. Should you ‘choose’ to throw in some family, health, or spousal issues, the mounting stress could be too much to handle. The truth is we have been coping with perpetual stress for so long, that often times we don’t realize just how much stress we are bearing. Too much stress in the medical profession has been associated with anxiety, depression, sleep problems, as well as poor academic and clinical performance. Stress can also decrease attention, concentration, and impair decision-making abilities which is critical to the delivery of appropriate care and pivotal to our profession. In extreme cases, stress can affect one’s ability to establish healthy relationships with peers and patients, resulting in feelings of inadequacy and dissatisfaction with clinical practice. Furthermore, such unresolved stress has been linked to suicide and drug and alcohol abuse in our professional community. Most universities and hospitals offer counseling services, mentorship programs, tutoring, and stress reduction workshops. However, data show these resources are vastly underutilized which suggest students develop his or her own methods to handle stress, which may be insufficient or even detrimental. Stress can be dealt with in healthy or unhealthy ways. Some of the unhealthy methods include drugs, alcohol, or other dangerous behaviors. The key to destressing is finding something you find relaxing or something you enjoy doing. What those activities are really depends on who you are and what you enjoy. A healthy escape will be a little different for everyone. For me, I love to exercise. So, after a long or stressful day, I look forward to going to the gym to blow off some steam. Some of my friends like to sit down and read, paint, or do some other hobby they enjoy. Others find meditation or yoga to be their route of stress relief. Whatever it is, find it, and try to incorporate it into your daily routine. We live in a world of deadlines and often feel we don’t have enough time in the day to do all the things required of us, much less finding time for ourselves. However, spending 30 minutes (right at 2%) of our day relaxing and learning new stress management skills is always time well spent, because of the psychological and physical health benefits it brings.
page 10
Keep in mind not all stress is bad. Small amounts of stress can actually be beneficial to you by sparking a healthy competitive spirit or bestowing the feeling of accomplishment after completion of a difficult task. As an example, remember the first time you aced a medical school exam, successfully utilized HVLA (especially cervical), or your first successful intubation? That feeling is good stress, or eustress, and can keep us motivated and enthused. Emergency Medicine is said to have one of the highest burnout rates of any specialty. To that end, do yourself a favor and start taking small steps everyday to reduce stress! You’ll be happier, healthier, and way more fun (read as moderately tolerable) to be around after a long shift. +
Tricks of the Trade By Chase Ungs, DO @ Doctors Hospital
Spring 2014
The Fast Track
A common encounter in the ED is a swollen finger with ring still present. Rather than simply cutting the ring, other options are available to remove the ring, including the â&#x20AC;&#x153;string technique.â&#x20AC;? 1. Patient comfort is important, a digital block may be provided if necessary. 2. Wrap a Penrose drain distally to proximally to exsanguinate the digit, remove the drain. 3. Using packing gauze, small string, or thick silk suture, pass one end underneath the ring. 4. Wrap the string clockwise around the finger along the entire length of the finger. If the string is not long enough, you may have to re-wrap the string later on. 5. The string should be wrapped tight enough to compress the swollen tissue, without tissue bulging between successive wraps. 6. Begin to unwrap the string on the proximal end, subsequently moving the ring distally as the string unwinds. The most difficult and painful area will be the proximal interphalangeal joint, care must be taken in this region. 7. Continue to unwind the string until the ring is removed from the finger. Contraindications to this technique include lacerations and fractures distal to the ring, as well as an uncooperative patient.
Reference: Stone, DB, Levine, MR. Foreign body removal. In: Roberts and Hedges Clinical Procedures in Emergency Medicine, 5th, Roberts, JR, Hedges, JR (Eds), WB Saunders Company, Philadelphia 2010. p.649-650. Image from Roberts and Hedges Clinical Procedures in Emergency Medicine
page 11
Pimpology 101 The Fast Track
Spring 2014
Top Things to Know on your EM Rotations Danielle Turrin, DO, MS @ Good Samaritan Hospital
T
he differential diagnosis for facial pain and swelling is sufficiently broad; however, we will be discussing the differences between periorbital cellulitis and orbital cellulitis. The primary reason to make the distinction between these two processes is that orbital cellulitis is a potentially life-threatening, ophthalmic emergency characterized by infection of the soft tissues behind the orbital septum. Periorbital cellulitis refers to the much more common and far less serious infection anterior to the orbital septum. It is possible for periorbital cellulitis to progress to orbital cellulitis â&#x20AC;&#x201C; although this is more likely in children. The first step to understanding the differences between these two diagnoses is to establish a basic understanding of the anatomy of the orbit and the orbital septum. The orbital septum consists of a layer of fascia extending vertically from the periosteum of the orbital rim to the levator aponeurosis in the upper eyelid and to the inferior border of the tarsal plate in the lower eyelid.
The bony orbit itself is made up of seven bones and contains the ocular globe, extraocular muscles, optic nerve, ophthalmic vessels, and the lacrimal gland. There are nine openings into the orbit consisting of foramina, fissures, and canals. These openings can serve as portals for infection to pass into or out of the orbit and into the surrounding space.
page 12
For more quick, easy, factual pimp-question information donâ&#x20AC;&#x2122;t forget to check our Pimpology 101 in our next issue of the Fast Track...
Similarities between periorbital and orbital cellulitis: • Fever • Eyelid edema • Pain • Red eye • Ill appearing
• • • • • •
Characteristics more specific to orbital cellulitis: Decreased eye movement and vision Diplopia Proptosis Globe displacement Pain with eye movement Chemosis (conjuntival swelling)
Spring 2014
The Fast Track
The floor of the orbit is the roof of the maxillary sinus, the medial wall of the orbit (lamina papyracea) is extremely thin & separates the orbit from the ethmoidal sinuses. The proximity of the orbit to the sinuses allows for potential spread of infection. The pathogens commonly isolated in both periorbital and orbital cellulitis include S. aureus, various Streptococcus species, H. flu (B type); and less commonly Eikenella, Klebsiella, Pseudomonas and Enterococcus.
Antibiotic therapy should be directed at such pathogens. Preseptal cellulitis may be treated as an outpatient with oral agents; whereas orbital cellulitis is generally admitted and treated with a course of IV antibiotics.
Pathophysiology of periorbital cellulitis: Pathophysiology of orbital cellulitis: • • •
Local skin trauma such as lacerations and bites Spread from local infection such as dacryocystitis, hordeolum and paranasal sinuses Spread from distant infections as well as from the upper respiratory tract
• • • • • •
Extension of infection from periorbital structures Paranasal sinuses (90% ethmoid) Also face, globe, lacrimal sac, dental infection Direct inoculation of orbit from trauma Hematogenous spread from bacteremia Occasionally an extension of periorbital cellulitis
The diagnosis of periorbital cellulitis versus orbital cellulitis is generally come to based on H&P, as well as CT scan of the orbits, sinuses and brain. Untreated periorbital or orbital cellulitis may advance to orbital abscess and/or subperiosteal abscess, which are far more serious and require more invasive treatments such as incision and drainage of abscess, and possible drainage by catheter placement within the sinuses. References Hunter, David G. MD, PhD & Michele Trucksis. “Preseptal (Periorbital) and Orbital Cellulitis.” UpToDate http://www.pediatriceducation.org/2005/12/ http://emedicine.medscape.com/article/1217858-overview
page 13
The Fast Track
Spring 2014
Perspective from an
EM LAWMAKER By Congressman Joe Heck, DO
W
hen I was elected to Congress in 2010, I was determined to retain the skills, knowledge, and tools I learned during my 25 years as an emergency medicine physician. My idea was to approach policy debates in Washington as a physician who happens to be a member of Congress, not a lawmaker who was once a physician. I believe bringing the perspective of a physician is key, not just to health care debates, but to all the major issues currently before Congress. Additionally, as physicians, your engagement with local and federal elected officials is crucial to shaping how your leaders think about those issues.
page 14
legislation helps me sort through all of the misinformation and grandstanding we see too often in Washington. As Osteopathic physicians we take a holistic, hands-on approach when diagnosing and treating patients. That approach has served me well when considering the numerous issues facing our nation. We know that structure influences function and that if one part of the body is experiencing illness, it is possible that other areas may be affected. The same is true here in Washington. None of the challenges we face exist in a vacuum. The problems, and often the solutions, are interconnected. We see this frequently in debates over economic policy. Since we cannot simply pass the “Improve the Economy Act of 2014,” we instead must find those interconnected microeconomic factors that can combine to improve the economy.
As Emergency Medicine physicians, we never know what is coming through the doors next or what is waiting behind the exam room curtain. Despite that uncertainty, we are always prepared to act. When we see a patient, we must first do a quick assessment - the first glance “sick” or “not sick” opinion. This is followed by a “history and physical” (H&P) and formulating a differential diagnosis (DDx). Next we must gather confirmatory On Capitol Hill, you can’t go very far without running into a committee or individual data, diagnose (Dx), and treat the patient. member working on legislation related to The same is true of my approach to health care. Health care has been at the legislating. When I first look at a bill, I try to center of American political debate since make a determination if this is a “good bill” the Patient Protection and Affordable Care or “bad bill.” I then read through it (H&P); Act was signed into law in 2010. Additionally, develop opinions (DDx); do research on the Medicare, Medicaid, and health care for issue (data gathering); take a final position veterans are all major policy areas that (Dx); and finally, vote (treat). I have found Congress is continually working on. It is using this systematic approach to analyzing critical that you, as physicians, engage with
your local and federal elected officials on these issues. There is no better advocate for a certain policy or piece of legislation than someone who is able to offer industryspecific knowledge of how that bill would help, or how it may cause problems. If state houses around the country are anything like Congress, there probably aren’t many physicians in office. This makes your opinion all the more important to these debates. Knowing how legislation will affect patients, physicians, and other health care practitioners is invaluable information to a public official who must formulate a position on a bill. Only you can speak from experience and there is no more powerful voice when you have the ear of a lawmaker. There are several debates currently happening in Washington that I encourage you to engage in right away. The first is the Sustainable Growth Rate or SGR. In my opinion, the SGR represents the single greatest threat to the Medicare system. Due to a lack of political will, physicians who treat Medicare patients are continually threatened with significant reimbursement cuts. I have been working in a bipartisan way to enact legislation addressing this issue but the wheels of change move slowly on Capitol Hill. Several House committees have taken more serious steps recently and hearing from local physicians might be just what we need to get a permanent Doc Fix across the finish line.
serious about reducing physician shortages, yet GME often comes up as an offset for other government spending. We must make a commitment to provide adequate resources to ensure that future generations of physicians receive the training and experience they need. You can lend your voice to this topic by telling Congress we need to ensure that the residency slots we currently have are allocated appropriately and that they’re funded adequately. Once we have a better picture of projected needs, we can begin to address the future and how we can add more residency slots to guarantee that we have an adequate supply of well-trained physicians ready to provide quality health care.
Spring 2014
The Fast Track
Finally, as emergency medicine physicians, I urge you to speak out about liability reform. Last year, the House voted to implement federal liability reforms based upon successful models like those of Nevada and California, but the measure was never considered by the Senate. This will continue to be an uphill battle in Congress but by you working with your state officials, we may be able to tip the scales in Congress and finally bring about meaningful medical liability reform.
By engaging with your local officials, you can bring more of the physician’s perspective to Congress and as emergency medicine practitioners, you know what it means to be prepared to act. Passing that knowledge and Another issue of grave concern is Graduate experience along will go a long way towards Medical Education. Politicians claim to be improving health policy in the U.S. + Dr. Joe Heck represents Nevada’s Third District in the House of Representatives. He is a 1988 graduate of the Philadelphia College of Osteopathic Medicine, completed his residency in Emergency Medicine at the Albert Einstein Medical Center of Philadelphia in 1992, and practiced emergency medicine in Las Vegas from 1992 to 2010.
page 15
The Fast Track
Spring 2014
By Greg Christiansen, DO, MEd, FACOEP-D @ Virginia Commonwealth University
A
55-year-old female presents to the emergency department for evaluation of palpitations. The palpitations started yesterday and are associated with familiar symptoms consistent with her prior diagnosis of paroxysmal atrial fibrillation. The EKG confirms atrial fibrillation with a ventricular rate of 130 beats per minute. The vitals are otherwise normal. Her examination is unremarkable except for the following findings: She has a midline chest scar and 2/6 systolic ejection murmur with a mechanical click suggestive of an aortic valve replacement. The patient confirms she had a mono-leaflet valve placed several years ago. She admitted to self-directed non-compliance with her Coumadin medication weeks ago due to a temporary issue with accessing her medication which was corrected on her visit. In looking at the clinical picture there are several concerns in addressing her disposition.
page 16
hours which is equivalent to fractionated heparin. There is an long term advantage with PO administration and if she becomes unstable, then cardioversion is an option. The patient needs additional studies including transesophageal echocardiography to identify clot formation.
C) Start oral rate control medication followed by oral anticoagulant like rivaroxaban (Xarelto). Additional aspirin therapy avoids the risk of stroke in atrial fibrillation and avoids INR testing associated with warfarin (Coumadin). This strategy allows the patient to be discharged for non-emergent outpatient studies at a lower cost of care.
Having trouble deciding which course of action to take? They all have reasonable considerations to address the conditions facing the patient. I had the same dilemma but I kept to my principle that all drugs are Which of the following decisions would you poisons if not used correctly. In this case favor as her treatment course and disposition? there is a real danger in choosing the wrong option. I needed to learn and recognize the A) Control her heart rate initially and admit risk and benefits of using a factor Xa inhibitor to an observation unit for administration of in this clinical condition. With this in mind I a fractionated heparin bridge and warfarin declined to offer a strategy until I had more (Coumadin) therapy. INR testing is required information. In particular, I was not familiar as is the addition of Aspirin therapy to limit enough with factor Xa inhibitors to know if potential stroke risk. they were safe in the setting of mechanical heart valves. I was aware of the indications of B) Admit to the telemetry unit for intravenous factor Xa inhibitors including: rate control, heparin bolus for bridging and Dabigatran (Pradaxa) therapy. Heparin â&#x20AC;˘ Reduced stroke and systemic embolism bolus immediately anticoagulates and risk in the setting of non-valvular atrial Pradaxa reaches full anticoagulation in two fibrillation
•
Treatment of DVT
•
Treatment of PE
•
Reduced recurrence of DVT or PE
•
Prophylaxis of DVT and PE in knee or hip replacement
Spring 2014
The Fast Track
Factor Xa inhibitors have been viewed as alternatives to warfarin and the RE-LY study showed factor Xa inhibitors were superior to warfarin in stroke prevention. However, in reviewing the literature and package insert, I came across the RE-ALIGN study which concluded factor Xa inhibitors should not be used to reduce stroke risk in mechanical heart valve patients. Factor Xa inhibitors have a limitation in the setting of mechanical heart valves. These drugs are direct thrombin inhibitors and act on the coagulation cascade at a single point rather than on the direct-contact thrombosis pathway. They cannot suppress the activation of the coagulation cascade once blood is in contact with an artificial surface. Warfarin on the other hand has three contact pathways to inhibit the thrombosis process. In the RE-ALIGN study the Dabigatran was associated with strokes, TIAs, MI’s, valvular thrombosis and increased bleeding incidence. The frequency was so alarming the study had to be terminated prematurely and a warning placed on the prescribing label. The Pradaxa package insert warns of the use of Pradaxa for the prophylaxis of thromboembolic events in patients with A-fib in the setting of valvular disease including mechanical or bioprosthetic valves is not recommended. In summary, remember this caveat: all drugs are poisons! Used judiciously in the correct dose and setting, the compounds are helpful in treating disease. You are not going to know the answer to every question, but you can educate yourself in what you are about to do. It is important to know the limitation of a medication before you administer the substance to a patient. Secondly, factor Xa inhibitors are generally useful medications in the correct setting in anticoagulating patients from thrombotic disease. If you chose scenario A, then you picked the safest course of action in this setting. Factor Xa inhibitors are not useful in the presence of artificial surfaces. Future implanted devices will have fewer mechanical surfaces and more biosynthetic surfaces so don’t count these drugs out prematurely. There will likely be new niches as advances in refining our understanding of the coagulation pathway moves forward. + References: Pibarot, Philippe, Dumesnil, Jean G. Valvular Heart Disease: Changing Concepts in Disease Management. Circulation 2009; 119: 1034-1048. Connolly S.J, et. al. Dabigatran Versus Warfarin in Patients with Atrial Fibrillation. New England Journal of Medicine. 2009; 361: 1139-1151. Gonsalves, W., Pruthi, R., Patnaik, M., The New Oral Anticoagulants in Clinical Practice. Mayo Clinic Proceedings. 2013; 88: 495-511. Eikelboom, JW, et.al. Dabigatran Versus Warfarin in Patients with Mechanical Heart Valves. New England Journal of Medicine. 2013;369:1206-1214. Dabigatran (Pradaxa) Package insert. Boehringer Ingelheim Pharmaceuticals Inc., April 26, 2013. Goldhaber, Samuel Zachery, Cardioexchange Editors. Dabigatran for Patients with AFib: Putting the Recommendations into Practice. NEJM Journal Watch. February 2011. www.jwatch.org
page 17
The Fast Track
Spring 2014
A
s a medical student interested in emergency medicine, I expect to learn how to resuscitate a patient presenting with diabetic ketoacidosis along with the associating physiology and pharmacology influencing this medical condition. Furthermore, I expect a solid foundation in all things pathology, physiology, anatomy and the list goes on and on. However, one such thing that seems to be lacking in the current state of medical education is law and politics. Outside of the medical education circle, we are engrossed with social media slandering about the current state of health care. However, many curricula do not find time to educate the students about the true law and politics. Medical ethics appears as one of the forefront topics being disseminated among students and residents, however one can argue ethics is in place solely to benefit and protect the patient, and even at that ethics is poorly taught.1 All of the medical education, and rightfully so, is about learning and treating disease to benefit the patient. However, with the dramatic alterations shaping the modern day medical landscape, physicians, residents and students should learn the basics of politics and law to benefit and protect themselves.
Law Politics and Education
According to a 2011 article in the New England Journal of Medicine, approximately 7.6% of emergency medicine physicians surveyed An editorial, by were sued.2 The article demonstrates Nicholas Pettit, Ph.D., OMS-II at OU-HCOM that while many physicians may be sued, few plaintiffs are actually successful with their lawsuit, illustrating the existence and prevalence of frivolous, unnecessary, meritless lawsuits. The stresses accompanying many physicians due to the potential lawsuit are unfathomable and can influence where the physician practices, what they choose to practice, and when they can retire. The litigious climate also goes back to affect the patient’s access to health care.2 Furthermore, the stresses resulting from a potential lawsuit not only reside with attending physicians, but also affect residents.3 Residents’ fear of malpractice and a potential lawsuit has a trickledown effect resulting in the practice of defensive medicine, which affect both the physician and the patient. The patient is typically burdened with unnecessary and expensive tests, meanwhile the physician has to order such tests to avoid an unnecessary lawsuit. The practice of defensive medicine is not only expensive for the entire system, but is also has a social implication whereby physicians have decreased quality of lives and unnecessary stressors. Lastly, a study from 2011 demonstrated emergency medicine physicians accumulate more stress, burnout more easily and intend to leave the specialty more often than physicians in other specialties.4 These articles paint a dim picture for emergency medicine. There is much research going into how to improve physicians’ quality of life, with such changes as the 80-hour rule during intern year, but I think some changes can be implemented in the actual formal education process. Like the integration of ethics in a medical school’s curriculum, which has demonstrated some effectiveness in physician compliance in dealing with ethical dilemmas, we need to be educated in situations beyond ethics to become a well rounded physician.5,6 However, learning only ethics leaves students clueless when non-ethical, law-based problems arise. The amount of ethics taught in medical school does not qualify one to become an ethicist, and similarly, students and residents need not be lawyers – they would only need to be introduced to areas like the business of medicine, medical malpractice, politics and the legal implications of practicing medicine in today’s era.7
page 18
One main issue to this argument is simply, when would a school integrate such lectures into their curriculum? One possibility is to include several lectures in conjunction with medical ethics to give a well-rounded understanding about law, ethics and politics to students. In addition to teaching ethics,
many schools now offer “unique” selling points that attempt to attract students to their college, such as problem based learning curriculum (PBL), early clinical exposure, and extra volunteer (and sometimes mandatory) experiences. Proponents of PBL have stressed it as the new wave of medical education, but some studies have shown that such methods of education may not produce differences once one is a physician.8 These aforementioned attractants, while both interesting and time consuming, have not been shown to create better physicians, increase physician happiness or frankly affect the overall quality of medical education (thus necessitating more future research). While there exists some research demonstrating that non-classroom based education (shadowing/volunteering) helps alleviate stress during medical school, there exists little research that non-classroom based education programs produce better, more competent and more caring physicians.9 It is clear medical schools are exceptional and even successful at including new ideas into their curriculum as demonstrated by PBL, ethics and shadowing, and thus I would challenge and encourage schools to adopt some law and politics into their curriculum.10 Some schools have begun to integrate courses within their curriculum, such as the business of healthcare. Such courses, however, are certainly not universal. Furthermore, like the introduction and development of ethics into medical school education, this appears to be an excellent opportunity for research to be performed in order to observe the effectiveness and utility in introducing law into medical education. Lastly, student doctors and residents need not be fluent in the language that is law, but for the betterment and advancement of our profession, it would at least be worth entertaining the idea of integrating law and politics into medical education. The goal of medical school and residency isn’t to produce lawyers, businessmen or law-savvy pedestrians, but it is to produce caring, compassionate and competent physicians. However, part of the medical educational process should benefit the physician, and by integrating law, politics and business I feel that physicians will be more multidimensional and better able to protect themselves and their profession. +
References 1. Pimentel D, Barbosa de Oliveira C, Vieira MJ. Teaching of medical ethics: students’ perception in different periods of the course. Rev Med Chil. 2011;139(1):36-44 doi:S0034-98872011000100005. 2. Jena AB, Seabury S, Lakdawalla D, et al. Malpractice risk according to physician specialty. N Engl J Med. 2011;365(7):629-36 doi:10.1056/NEJMsa1012370 3. Rodriguez RM, Anglin D, Hankin A, et al. A longitudinal study of emergency medicine residents’ malpractice fear and defensive medicine. Acam Emerg Med. 2008;14(6):569-573 doi:10.1197/j. aem.2007.01.020 4. Estryn-Behar M, Doppia MA, Guetarni K, et al. Emergency medicine physicians accumulate more stress factors than other physicians-results from the French SESMAT study. J Emerg Med. 2011;28(5):397-410 doi: 10.1136/emj.2009.082594. 5. Goldie J, Schwartz L, McConnachie A, et al. The impact of three years’ ethics teaching, in an integrated curriculum, on students’ proposed behaviour on meeting ethical dilemmas. Med Educ. 2002;36(5):489-497 doi:10.1111/j.1365-2929.2004.01915.x 6. Shah N. The teaching of law in medical education. Virtual mentor. 2008;10(5):332-337 http://virtualmentor.ama-assn.org/2008/05/oped1-0805.html Accessed Janurary 20, 2014 7. Studdert DM, Mello MM, Sage WM, et al. Defensive medicine among high risk specialist physicians in a volatile malpractice environment. JA MA. 2005;293(21):2609-2617 doi:10.1001/jama.293.21.2609. 8. Hartling L, Tjosvold L, Oswald A. Problem-based learning in pre-clinical medical education: 22 years of of outcome research. Medical Teacher. 2010;32(1):23-35 doi:10.3109/01421590903200789 9. Holm M, Tyssen R, Stordal K, Haver B. Self development groups reduce medical school stress: a controlled intervention study. BMC Medical Education. 2010;10(23) doi:10.1186/1472-6920-10-23 10. Mattick K, Bligh J. Teaching and assessing medical ethics: where are we now? J Med Ethics. 2006;32:181-185 doi:10.1136/jme.2005.014597
Spring 2014
The Fast Track
page 19
The Fast Track
Spring 2014
MASTER THE BASICS:
Fluid Management by Robert Culley, OMS-III @ WVSOM
We have all struggled
with the idea of fluid management. It is a seemingly simple concept, yet many of us are baffled by the thought of choosing which fluids to give and how much. While there are some concepts that are quite difficult in fluid management, here are some basics that may help. The body is broken down into conceptual compartments holding different proportions of total body fluid, as seen in the figure on page 22. The majority of fluid is found within the intracellular compartment, and the remaining within the extracellular compartment. The extracellular component is broken down into the interstitial space and intravascular space. The intravascular space is by far the smallest compartment, yet can have the most impact on the body. This is the compartment maintaining perfusion to all other tissues, and is the compartment which we can easily determine if it is lacking in fluid. Now we can take a look at fluid intake where the average person has roughly 1-1.5 L fluid intake per day. The urine excretion consists of 0.5-1 L per day, but there is insensible fluid loss as well with about 0.4 L lost from the lung per day, 0.5 L of fluid lost in sweat and 0.2 L in stool. These numbers change with fever and activity.
Next, we have plasma osmolality where 2(Na+) + (BUN/3) + (Glucose/20) = 275-290 mosm/kg. If we have a patient that has normal glucose and BUN, we can essentially ignore them and say the osmolality is equal to two times the sodium level of the patient. There are numerous types of fluids, and each has its own role in fluid management. We first have the crystalloids: dextrose in water, saline, combinations of the two, and Lactated Ringer’s. Each of these fluids are broken into different solutions and concentrations, with Lactated Ringer’s being the most “physiological.” Next we have Colloids, which are proteins in solution: albumin, dextrans, and hetastarch. And lastly, we have blood products including whole blood, packed red blood cells (PRBC’s), fresh frozen plasma (FFP) and platelets. Each of the aforementioned fluids have a specific indication and reason to utilize them. We’ll introduce them with cases: A 64-year-old female arrives by ambulance to the ED. She has a past medical history of diabetes mellitus type II and history of DVT. She has been on Warfarin treatment for four months now. Recently she has been complaining of easy bruising and has black stools. The patient has an INR of 12 and a hemoglobin of 6.7 g/dL. This patient will need treatment with vitamin K, discontinuing further Warfarin treatment and possible fresh frozen plasma (FFP). We all have some ideas as to when to give certain products. For example, FFP can be given to a patient with an active bleed who has a supratherapeutic INR on Warfarin and PRBCs can be given to a patient with a hemoglobin ≤7.3 These are simply two straightforward examples, but this is not where many of us struggle with fluid management.
page 20
Crystalloids have become the mainstay of fluid management, with their use being frequent and their cost low, roughly $3. Saline solutions are the most commonly used type of fluid. These
solutions make up the majority of what we use in the emergency department, and there is a very important reason as to why we use them so prolifically. It is all about plasma osmolality and what compartment the fluid will resuscitate. A typical case might be a 31-year-old male arriving by ambulance after being discovered in an alley near an apartment building. The patient is responsive to verbal stimuli. His blood pressure is 85/58 and has a heart rate of 134 bpm. This patient needs fluid resuscitation of the intravascular compartment as he is symptomatic for fluid loss within this compartment. Although he will need other treatments, it is important to recognize the need to treat for the intravascular symptoms of tachycardia and hypotension.
Spring 2014
The Fast Track
The average person’s osmolality is roughly 280-300 msosm/kg (2 x Na+ level). This is very important because a solution of isotonic (normal) saline of 0.9% NaCl has an osmolality of 300 osm. This means that normal saline has roughly the same osmolality as our blood. So we can now half that solution to the hypotonic 0.45% NaCl or ½ normal saline, and this has an osmolality of 150 osm. We then look at the hypertonic 3% NaCl with an osmolality of 514 osm. Once you see these numbers it becomes relatively clear why we utilize normal saline (0.9% NaCl) so often in our treatments. Osmolality is only one aspect of fluid management, with another part being where the fluid goes once infused into a patient. With saline solution, in particular 0.9% Na+ solution, all of the fluid stays within the extracellular space. Roughly 75% goes into the interstitial space, and the remaining 25% goes into the intravascular space - illustrated by the diagram on the next page. This means in a hypotensive, tachycardic patient with an active bleed whom you gave a liter bolus of NS, only 250 mL of that fluid remained in the patient’s intravascular space and begins to resuscitate that patient’s intravascular volume and hemodynamic status. Something like whole blood, however, will remain almost entirely intravascular. But it is costly, limited, and has substantial risks to administration. A 20-year-old male is brought in by his parents after running a marathon for charity. The patient is weak but is ambulating and talking without difficulty. The patient looks thin, his mucous membranes are dry and complains of being very thirsty. His blood pressure is 118/79 and heart rate is 62 bpm. The patient’s urine is dark. This patient is asymptomatic in the intracellular space and has a deficit of pure water. He needs to have volume replacement but it is more important to resuscitate the intracellular space and work on his dehydration. Dextrose solutions have significantly different purposes than their saline counterparts. We have varying concentrations of dextrose: D5W, D10W, and D50. D50 is used to raise glucose is someone who is hypoglycemic. D5W is used mainly in someone who is hemodynamically stable and is dehydrated - i.e., with a deficit of pure water. This is mainly due to where the fluid goes in treatment. In these solutions, the fluid travels mostly into the intracellular space, with roughly 25-75 mL out of 1000 mL, remaining in the intravascular space. So if the D5W is utilized in a patient who is hemodynamically unstable needing intravascular fluid resuscitation, we can see that it will not do a very good job. If that same patient was stable and was simply dehydrated, meaning the fluid within the intracellular and interstitial space was low, then D5 is the best choice for fluid resuscitation. The trace amount of dextrose in D5W is there purely to prevent osmotic cytolysis near the infusion site. page 21
The Fast Track
Spring 2014
Fluids with protein are some of the newest creations within the field of medicine. These fluids are so new, that many hospital systems rarely carry them. They are extremely expensive, but work well. There are quite a few uses for which they are intended, but here we will only discuss their role in fluid resuscitation. Protein solutions can offer fluid management for hemodynamically unstable patients, but there has been no definitive evidence over utilizing crystalloid solutions. Utilizing these protein solutions, you can get up to 750 mL out of 1000 mL delivered within the intravascular space, which would give the largest resuscitation of the intravascular space from any of the fluids mentioned. This does not mean that it is the best choice as evidenced by current studies and the possibly of side effects such as anaphylaxis and acute renal failure.1 The last type of fluid to discuss is Lactated Ringer’s, which is a solution of varying electrolytes placed in specific concentrations to mimic human plasma. Lactated Ringer’s is an isotonic solution generally used to replace fluids during operations. It provides roughly 100 mL of free water into the intravascular space per 1000 mL.4 This solution is of mixed use within emergency medicine, as other solutions can be used with the same efficacy and are less expensive. As it has less an effect on the intravascular space than normal saline solutions, Lactated Ringer’s solution is not often used in the ED. In emergency medicine the importance lies within the resuscitation of the patient, whether intravascularly or intracellularly. These two actions have crystalloid solutions that can be better utilized for this use, and Lactated Ringer’s solutions are less suitable for resuscitation in emergency cases. Lactated Ringer’s is seen in the operating room and in places where fluid management calls for large volumes such as medicine delivery on floors.4 Within the operating room, this solution is of most benefit as the patient is receiving more than two liters and there is a need to maintain physiologic conditions. Lactated Ringer’s can cause some hyponatremia at large volumes, but this effect is minimal. It must also be held in patients with lactic acidosis.2 Making decisions about when to use various fluids can be difficult at first, but as time goes on the tricks and nuances are easily acquired. There are quite a few other details regarding fluids and fluid management, but the basics were presented in a way that can hopefully be utilized without any problems. Cost plays a large role in choosing fluids as well as availability. It is often the cheapest and most available resource that is utilized the greatest, and this is seen with the copious use of crystalloid solutions. +
page 22
References: Godara, H. (2014). THE WASHINGTON MANUAL OF MEDICAL THERAPEUTICS (34th). Philadelphia, PA; Lippincott Williams & Wilkins . Retrieved March 1, 2014 from http://my.wvsom.edu:2147/Resource/Title/145118851X McMillan, J. & Feigin, R. & DeAngelis, C. & Jones, M. (2006). OSKI’S PEDIATRICS (4th). Philadelphia, PA; Lippincott Williams & Wilkins . Retrieved March 1, 2014 from http://my.wvsom.edu:2147/Resource/Title/0781738946 Gabrielli, A. & Layon, A. & Yu, M. (2009). CIVETTA, TAYLOR, & KIRBY’S CRITICAL CARE (4th). Philadelphia, PA; Lippincott Williams & Wilkins . Retrieved March 1, 2014 from http://my.wvsom.edu:2147/Resource/Title/0781768691 Morgan, G. & Mikhail, M. & Murray, M. (2006). CLINICAL ANESTHESIOLOGy (4th). New York, NY; McGraw-Hill Companies. Figure from http://www.frca.co.uk/article.aspx?articleid=289
Spring 2014
The Fast Track
Didnâ&#x20AC;&#x2122;t get to present your case in April?
KEEP CALM AND
JOIN THE COMPETITION Present at the Fall Conference in LAS VEGAS!
FOR MORE INFORMATION VISIT
WWW.FOEM.ORG THE 4 COMPETITIONS ARE: Oral Abstract sponsored by EmCare, Research Paper, Research Poster sponsored by Florida Emergency Physicians
and the CPC Competition sponsored by Schumacher Group
page 23
The Fast Track
Spring 2014
The View from the Bottom: An internâ&#x20AC;&#x2122;s look at how things really were and are..... By Brian Lehnhof, DO PGY-1 @ Kent Hospital
T
his is always an exciting time of year. Congratulations to all those that matched into EM; welcome to the family. It will be a pleasure to have you join the ranks of interns.
In preparation for the changes that will come in each of our careers as we advance an academic year, I stop and reflect on the past year and reminisce on the experiences I have had. Intern year is designed to stretch and test us; it comes with a lot of stress, sacrifice and sorrow. Right along with these negatives come the innumerable lessons and experiences that are formative to who we each become during our medical career. However, intense retrospection is of little value if it is not tightly coupled with prospection. The strongest people I have come to know are those who are able to look inward and identify where their weaknesses and vulnerabilities lie and convert them into their strengths. This is a process that can be achieved only with retrospection and subsequent prospection. Since my youth in Southern California, I have enjoyed surfing. Now that I again reside on a coastline, I am able enjoy the creations of the Earthâ&#x20AC;&#x2122;s tides once again. Recently, I paddled out into the frigid New England water and observed an interesting lesson. It is well known that the surf is consistently better during the winter months; this can become problematic. When I lived in Hawaii, the change of season was almost negligible except for the change in the surf. Now living in New England, the change is anything but subtle. In order to take advantage of the winter swells of the northeast, I have to make distinct preparations. If I wear the right wetsuit with the accompanying gloves, boots, and hood I can be quite comfortable in an otherwise harsh and hostile environment. My simple preparation allows me to accomplish what was previously not possible due to impending hypothermia.
page 24
My winter surfing predicament is not unlike the ones we encounter in medicine. Entering the medical field is a big step and a daunting task. Like the frigid Atlantic Ocean, the hospitals we work in can be a challenging place to be immersed. Our work environments
“
All of the effort, struggles and sacrifice are WORTH IT.
Spring 2014
The Fast Track
“
are intense and at times overwhelming. We do not start our careers equipped with all the skills and strengths that are needed to survive all the scenarios we encounter. Careful preparation allows us to gain control and confidence in the uncomfortable settings we are consistently found. A great man once said that “...if you are prepared you do not have to fear.” This directly applies to our life in medicine, and in particular emergency medicine. Being an intern allows me to be exposed to so many different situations and scenarios. Each of these is coupled with interactions with attendings and colleagues who have their own distinct and unique perspectives. I will be the first to admit that I often question the utility and relevance of some off-service rotations; however, without fail I always come away from these experiences with more strength and knowledge than what I went in with. These experiences collectively add to who I am and better prepare me to thrive in the ED. On any given day as an intern, I fail at something. Most days I fall short at many things. Perhaps I botched a procedure or missed a diagnosis. Whatever the issue, it gives me a chance to reflect on how I would perform if given the chance to do it again. It motivates me to research the missed diagnosis, review the failed procedure, or to do whatever else is needed to prepare for when the situation presents itself again. We are all going to make mistakes and we are going to fall short. The best aspect about our training is that we are exposed to things in a repetitious manner. Pathology and circumstances repeat and we will experience things on multiple occasions. The challenge is to identify when we fall short, learn from these experiences and do better the next time. My challenge to all of you, no matter where you are in your training, is to always reflect on how you are performing. Look inward. Try to find something each day that you could have done a little differently. Review what went wrong; decide what could have been better. Visualize the experience happening again, but this time with the new and improved ending. This will help to bring closure to bad a outcome or a negative experience. This will make you a better clinician for your next shift when the whole thing happens again. So long for now, keep up the good fight. All of the effort, struggles, and sacrifice are worth it. People truly do depend on what we do, even if we don’t do it perfectly. +
page 25
The Fast Track
Spring 2014
Being Politically Active
by John Casey, DO
How Do Politics Apply to EM?
I
often talk with students and residents about what they feel they need to learn more about, or have more education in. The usual responses I get include medical topics like splinting labs or social needs such as contract negotiation and debt repayment. Sadly, I have yet to hear someone say “political action.” This is unfortunate because every other topic in healthcare is underpinned and driven by legislation. Not being politically involved is the equivalent to knowing someone is in V-Fib and looking the other way because it’s some other intern’s job… it’s just not a good way to practice emergency medicine. To illustrate my point, I thought I would let you wander around in my skull a bit during a shift (it’s a scary place) – so that you see why political involvement is so important for the future of emergency medicine.
page 26
He wrecked his bike getting onto the interstate about five hours before he came to the ER. He’s elderly, and didn’t really want to come in but finally decided he was hurting too much. He is very tender over the left chest wall and has bruising of his left foot. After imaging, he is found to have a fractured foot and multiple rib fractures. I’m at a small community hospital, so I contact our
local trauma center to arrange transfer. I do EMTALA paperwork and appreciate the fact that the interstate he wrecked on is the reason My first patient of the shift is we have an integrated EMS a motorcycle crash victim. system to bring the patient
into the hospital (the federal government used federal interstate funds to “nudge” states into developing EMS systems). My second patient of the night is a regular. Her number of visits for dental pain this month exceeds her actual number of teeth, so I’m not quite sure why she’s here again. Actually, I’m fairly certain I know why she’s here again. Regardless, I’ll give her a thorough screening exam and stabilizing treatment to the best of our facilities’ capabilities. At this moment I’m rethinking the importance of EMTALA. Before I go see my next patient, my phone blips to let me know a “transfer of funds exceeding my preset notification limit” has been sent to the federal government. Now I’m thinking about student loan repayment. That brings me to SGR reform, student loan reform and the soaring costs of medical school. Not to mention the fact that our physician graduates exceed the number of residency
The Fast Track
CapWiz: http:// corporate.cqrollcall.com/ content/355/en/Capwiz
Spring 2014
Advocacy Resources:
AOA Grassroots Campaign: http://www.osteopathic.org/ inside-aoa/advocacy/Pages/ goal-video.aspx
training spots. Somebody I challenge you to take better get on that. just 10 minutes a month (you read that right) to be It turns out that that involved. This will put you somebody is me... And you! ahead of what most of If you are hoping someone your peers and practicing else is going to fix these physicians are already doing. problems, you’re actually Set an appointment on your right. The federal government calendar to remind you to do can always come up with a it. Email your representatives solution… it’s just usually not in the local, state, or federal one that benefits any of the government about a topic you parties involved. If you have an interest in reducing your care about. Read a political student loan debt, making article with an opposing view sure you get paid fairly for and draft a letter to the editor. the work you do, protecting Make a public comment the transfer of patients, on a piece of policy that regulating the distribution is up for debate. The only of controlled substance or way to be empowered as a really any other aspect of physician is to take action. medicine, NOW is the time If you aren’t willing to be a for you to get involved. You part of the solution, then you will never have more time, so are absolutely a part of the problem. + just learn to do it now.
Dr. Casey is an attending at Doctors Hospital in Columbus, OH where he is also core faculty for the OUHCOM/Doctors Hospital emergency medicine residency. He is actively involved in simulation based education, EMS and political advocacy.
When Dr. Casey was a chief resident in 2012, he was honored by the White House as one of its “Champions of Change” for facilitating positive change and helping the community understand the Affordable Care Act. Read more here:
page 27
â&#x20AC;&#x153;
The Fast Track Many of the representatives I met with were familiar with
Spring 2014
the bills we discussed and were as frustrated as we were at the lack of progress being made. Overall, it was an inspiring day. Witnessing hundreds of white coats on Capitol Hill was a sight to be seen. For those looking to become more involved, the AOA has an advocacy section on their website (www. osteopathic.org). For residents interested in health policy, there is the TIPS (Training in Policy Studies) fellowship that
Most notably for me, being
can be completed during the 3rd or 4th year of residency. It
cess made the issues more pe
is run by NYIT and offers extensive training in the political
in DC, I not only became more
process. More information can be found on the website
obstacles, I became more awar
http://iris.nyit.edu/nycom/tips/.
thing about it.
~Denzil Frost DO, Charleston Area Medical Center
~Michelle Kinghorn OMS II, Arizon
DO DAY
What I took away from this exp
to become an advocate for the
represent the osteopathic com
and advocacy opportunities. O from us! Having never attended DO day on Capitol Hill, I had no idea the galvanizing impact this experience would have toward my goals of becoming a physician, a leader in my community, and a political advocate for my community. page 28
~John Cloud OMS III, Oklahoma State University COM
~Jaclyn Ronovsky, ACOEP Sr. Coor
Spring 2014
The Fast Track
I realized just how important it is to actually take part in an activity of this nature instead of just watching by the wayside. I have no doubt that the 1200+ students and a part of the lobbying pro-
ersonal.... During my stay
attendings that showed up for DO Day made a significant impression
e aware of these potential
and will help shape the political
re that we could do some-
landscape in some way.
na COM
~Nick Blair OMS III, ATSU-SOMA
on the hill Perspectives
perience is how important it is
e health care profession, and to
mmunity through health policy
Our representatives want to hear
rdinator Member Services
â&#x20AC;&#x153;
And although the news would like to make us think that Washington is in a stalemate, maybe DO day was a sign that good things are to come. ~Andy Little DO, Doctors Hospital
page 29
The Fast Track
Spring 2014
Managing Suspected Appendicitis By Esther Hwang, OMS-III @ LECOM
Imagine
yourself as a first year emergency medicine intern, or a medical student in the ER, doing an overnight shift. You are told there are three new patients that need to be seen:
An 8-year-old female presents with vomiting and RLQ abdominal pain for the past two days. Her parents initially believed she had some sort of stomach bug. Today, her abdominal pain became diffuse, and she then spiked a fever of 102oF which has not been relieved with children’s Tylenol. When you approach her at bedside, she is curled up in the hospital bed, saying in a small voice that she is most comfortable that way. While you try to have her lie on her back for an examination, she is visibly uncomfortable and distressed despite the fact she is trying not to cry. A stressed out, 27-year-old, graduate student presents with abdominal pain that started initially as what she believed was heartburn about a day ago, but it has traveled from her periumbilical region to her right lower quadrant. She states that she thinks she may just be “backed up,” but her roommate thinks otherwise. She is afebrile. She keeps asking when she can be discharged so she can go home and continue studying. An elderly, 79-year-old man with a history of dementia is brought in by his daughter. He had been mumbling about low stomach pain, pointing to his right side. He also says he is having some urinary frequency and urgency. He has refused to eat for the past two days and his daughter decided that he ought to see a doctor about these symptoms. At his primary care physician’s office, his doctor advised them to go immediately to the emergency room.
DDx
What are the differential diagnoses for these patients? Is it easier to immediately suspect appendicitis in the 8 year old or 27 year old? Would you even include appendicitis in the differential for the elderly patient? What are the clinical presentations of appendicitis for each age group and do they differ from gender and age categories?
“The Patient Presents with Abdominal Pain…” The National Health Statistics Reports published in 2006 rank the chief complaint of abdominal pain as the most common reason that patients sought treatment in the emergency rooms nationwide.1 The importance of rapidly recognizing abdominal pain as an emergent situation versus a more innocuous diagnosis is crucial in any good emergency medicine education. Any of these patients above could potentially present at any emergency department and it is up to the medical team to be able to determine rapidly whether it is simply harmless abdominal pain, or if it is something that needs to be further observed or if surgical consultation should be involved as well. A surgical emergency, appendicitis appears routinely in emergency departments nationwide. Despite the ability to diagnose appendicitis based upon clinical presentation, the patient’s age, gender and body habitus are among some factors that can cause significant variations in the patient’s presentation. Appendicitis has a lifetime occurrence of 12% in men and 23% in women.2 Definitive treatment is surgical intervention and if left untreated, appendicitis can cause serious complications such as bowel perforation, peritonitis, sepsis and even death.3
page 30
Clinical Appendicitis Appendicitis is inflammation of the appendix, which is located at the cecum, near the ileocecal junction
where the small intestine becomes the large intestine, as seen in the figure below. The clinical picture of a classic case of appendicitis presents with the subjective findings of periumbilical pain that migrates to the right lower quadrant with associated nausea, anorexia and fever. Physical findings include confirmation of the fever and peritoneal signs such as rebound pain at McBurneyâ&#x20AC;&#x2122;s point. Laboratory findings include leukocytosis and a left shift of white blood cell count. These findings actually make up the Alvarado Score, a diagnostic tool that stratifies the risk and likelihood of appendicitis in patients who present with abdominal pain.3,4
Spring 2014
The Fast Track
The Alvarado Score Created in 1986, the Alvarado score uses a 10 point clinical system where scores range from 0-10. An Alvarado score of 1 - 4 usually routinely predicts that the patient has a very low likelihood of having appendicitis. Scores of 7 or higher will have a much higher likelihood that the patient has appendicitis. Variations in gender and age have been identified as significant factors that can change the accuracy of a diagnosis.5 Despite advancing the standardization of acute appendicitis diagnostic criteria, Alvarado scoring has been debated for its effectiveness as a diagnostic tool. The difficulty in diagnosing appendicitis from a clinical point of view is when the Alvarado score falls in the equivocal range, which is between the values of 4 - 6. In a retrospective study in 2004 of medical charts at a northwestern Pennsylvania acute care facility, 21/69, or 35.6% of patients were correctly diagnosed with appendicitis with just the equivocal Alvarado score. In comparison, this study found that patients with a score of 7 or higher had a 77.7% incidence positive rate for appendicitis. 6 Different subsets of populations can result in a broad range of variability in clinical presentations. In women, gynecologic pathologies have to be considered in the differential diagnosis, whereas in children, signs like fever seem to take precedence in light of other tell tale signs such as RLQ pain.6,7 The physiological effects of aging in the elderly can alter the presentation of appendicitis. Diseases that can affect the mental clarity of an elderly patient as the introductory clinical vignette of the 79 year old shows, can make taking a history of the course of illness difficult. Although the classical clinical presentation is still tell tale in the elderly, multiple comorbidities including atherosclerosis, decreased circulatory flow and increased risk for obstruction due to decreased peristaltic action can mask the classic symptoms.8 Imaging To reduce the risk of both over or under diagnosing appendicitis based on clinical and laboratory findings, computed tomography has become a standard in diagnostic imaging and has reduced the occurrence of a negative appendectomy to less than 10%.6 The most common identifiers on CT, raising the suspicion of acute appendicitis, are fat stranding and dilation of the appendix.9
Dx
With acute appendicitis, CT has been the most accurate means of imaging for patients with an unclear clinical presentation of the illness. Compared to ultrasonography, CT surpassed US in both sensitivity and specificity at 91% versus 78% and 90% versus 83%, respectively.10 So why would it be inappropriate to order a CT for every patient that presents to the emergency room with abdominal pain
page 31
The Fast Track
Spring 2014
suspicious of appendicitis when CT scans are fast and the image generated can be read within minutes? Abdominal CT imaging exposes a patient to ionizing radiation of 8 mSv approximately, whereas a chest X-ray exposes patients to the equivalent of 0.02 mSv and a lumbar spine X-ray to 1.5 mSv of radiation.11 While the average lifetime risk does not increase with a single imaging exposure, there has been concern raised with using a CT as an initial diagnostic tool especially in pediatric populations due to the exposure of ionizing radiation.4,10,11 In addition, there is additional prep time of over an hour when contrast dye is added to enhance the imaging. Moreover, there has been no reported significant increase of accuracy with the addition of enteric, IV or oral contrast dye though it is still preferred by most radiologists.6,10 If a patient is in fact presenting with an acute surgical belly, such as a perforated appendix, the time waiting for the procedure also increases the patient’s length of stay in the hospital. Therefore, how exactly is it possible to determine which patients require additional imaging, and which patients should, in essence proceed directly to surgery? Putting it all Together The American College of Radiology and the American College of Emergency Physicians both recommend evidence-based appropriateness criteria for different health conditions and age groups that delineate the necessity of certain imaging techniques based upon clinical presentation. The criteria presented by the ACEP and the ACR are based upon recent data analyses and evidence of risk or benefits of imaging studies. Both institutions have established guidelines that use risk stratification for the necessity of diagnostic imaging. The ACR discusses the benefits and risks of multiple imaging modalities available for diagnosing acute appendicitis, which includes the ever favored CT, ultrasonography, X-ray, MRI, and the Tc-99m WBC scan.10,12 It is implied that the “classic clinical presentation” scores high on the Alvarado scoring, which includes signs such as migratory RLQ pain, rebound tenderness, and anorexia.5 The ACEP and the ACR recommend that while appendicitis may be able to be diagnosed clinically, imaging techniques increases the sensitivity and specificity of the diagnosis. For patients with atypical presentation, both the ACEP and the ACR suggest that CT is still the most appropriate imaging modality, especially when clinical presentation is inconclusive. However, the ACR also recommends that abdominal X-ray and ultrasound may also be recommended initially for certain age groups, and in order to exclude other etiology of abdominal pain such as free air or obstruction.10,12 While meta-analysis has demonstrated the CT is superior in diagnostic imaging, considering the risks, the patient’s age and predisposing medical conditions, including pregnancy status in females and allergies, different imaging procedures are still recommended. Especially in children younger than 14 with atypical presentation, and pregnant patients, US is recommended as the first step in imaging. CT is then performed if the US is inconclusive. The caveat of ultrasonography is that it is user specific and the sensitivity of US accuracy differs greatly depending on the skill of the individual user.10 Next Steps Now let us return back to the three patients presented in the introduction. Based upon the history obtained, you have concluded that there is a high suspicion for appendicitis for all three patients. You have given the child an Alvarado score of 10, the graduate student a 7, and the elderly patient’s score has remained inconclusive since he and his sister are unable to convey a thorough history to you.
Tx page 32
What would you do next? Among the sequence of events after your differential has been determined, is to quite possibly wake up surgeons and notify them that there are three new surgical consultation requests due to your high suspicion that each patient is in need of emergent surgery. The justification
for this consult must be based off evidence results, and whether or not imaging is to correctly utilize the diagnostic criteria a clear picture to the surgeon as to why he
from the history taken, the physical exam and laboratory warranted. This is why the importance of knowing how available can enable a student, intern or resident to present or she strongly believes that case could be appendicitis.
Spring 2014
The Fast Track
At this juncture, the importance of a knowledgeable history and physical plus the awareness of the ACR and ACEP guidelines can enable you to be able to present your recommendation to the surgeon. In the 8-year-old girl’s case, you present to the attending physicians that you strongly believe that based off her history of present illness and her laboratory results, you believe without a doubt that this is a case of a ruptured appendix with suspected peritonitis. You reason to them that imaging could possibly delay her time to surgery and further worsen her condition. During your patient interview, the 27 year old admits that she is sexually active. She admits her menstrual cycle is irregular, so you obtain the routine pregnancy test since she is of childbearing age. Her subsequent negative pregnancy test allows for additional imaging with CT, and you also determine that the elderly 79-year-old patient could also benefit from CT imaging as well. The patients are subsequently imaged, and appendicitis is confirmed in both. Communication is Key Managing acute appendicitis is a multidisciplinary practice in medicine. Time is truly of the essence when faced with abdominal pain suspicious for appendicitis. Oftentimes, the emergency physician is the initial point of care for the patient with suspected appendicitis. It is at this juncture where the course of action to mobilize the rest of the medical parties may or may not occur based off of the emergency department’s presentation. An emergency physician’s proper history and physical, coupled with knowledge of the Alvarado score and the criteria set forth by the ACR and ACEP to proceed or bypass with diagnostic imaging enable efficient and sound communication strategies to present to the surgical team. This enables the surgical team to be able to make a rapid decision especially in the face of emergency surgery. + References
1. Pitts SR, Niska RW, Xu J, Burt CW. National Hospital Ambulatory Medical Care Survey: 2006 emergency department summary. 2008. National health statistics reports no 7. Hyattsville, MD: National Center for Health Statistics. 2. Addiss DG, Shaffer N, Fowler BS, Tauxe RV. The epidemiology of appendicitis and appendectomy in the United States. Am J Epidemiol. 1990;132:910–25 3. MedlinePlus [Internet]. Bethesda (MD): National Library of Medicine (US); [updated 2014 Feb 28]. Appendicitis; [updated 2013 Jul 18]; Available from: http://www.nlm.nih.gov/medlineplus/ency/article/000256.htm 4. Armstrong C. ACEP releases guidelines on evaluation of suspected acute appendicitis. Am Fam Physician. 2010, Apr 15; 81(8):1043 – 1044 5. Alvarado, A. A practical score for the early diagnosis of acute appendicitis. Ann Emerg Med. 1986, 15:557-564 6. McKay R, Shepherd J. The use of the clinical scoring system by Alvarado in the decision to perform computed tomography for acute appendicitis in the ED. Am J of Emerg Med. 2007, 25:489-493 7. Ohle R, O’Reilly F, O’Brien K, Fahey T, Dimitrov B. The Alvarado score for predicting acute appendicitis: a systematic review. BMC Med. 2011, 9:139 8. Garba S, Ahmed A. Appendicitis in the elderly: Appendicitis - A Collection of essays from Around the World. Intech. 2012, Lander A (Ed.), Available from: http://www.intechopen.com/books/appendicitis-a-collection-of-essays-from-around-the-world/appendicitis-in-the-elderly 9. Old J, Dusing R, Yap W, Dirks J. Imaging for suspected appendicitis. Am Fam Physician. 2005 Jan 1;71(1):71-78 10. American College of Radiology. ACR appropriateness criteria: right lower quadrant pain – suspected appendicitis. Available at: http://www.acr.org/~/media/7425a3e08975451eab571a316db4ca1b.pdf 11. Taylor G, Wesson D. Acute appendicitis in children: diagnostic imaging. UpToDate Inc. N.p., 25 [Updated 2011 Jan]; Available at: http://www.uptodate.com/contents/acute-appendicitis-in-childrendiagnostic-imaging?source=search_result 12. Howell J. Clinical Policy: Evaluation and management of suspected appendicitis. American College of Emergency Physicians. [updated 2010 Jan]; Available at: https://www.acep.org/Clinical---PracticeManagement/Clinical-Policy--Evaluation-and-Management-of-Suspected-Appendicitis/ Image from http://www.nlm.nih.gov/medlineplus/ency/presentations/100001_1.htm
page 33
The Fast Track
Spring 2014
of the most overlooked leadership positions for osteopathic medical students is serving as a Student Representative on AOA’s bureaus, councils, and committees. The B/C/C’s cover every aspect of health care, from health policy to education. By applying, students will have the opportunity to “work on a national level representing the views of osteopathic medical students and DO’s in postdoctoral training.” Through this experience, students can work directly with AOA leadership and contribute to both the AOA and their own professional development. As the 2014 Student Representative on the Bureau on Federal Health Programs, I had the amazing opportunity to do just that. What follows is a reflection on my experiences and a brief peek inside our meeting this past January. As I take my seat between the other members of the AOA Bureau on Federal Health Programs (BFHP) at the conference room of the organization’s Washington, D.C. headquarters, I realize how out of place I might appear. My business attire provides little in the way of blending in, though I’m certain it must look quite sharp on me. Despite my awkward appearance, I locate my seat and take a second to survey the room. The Bureau members are seated side-by-side, acting as a panel for the day’s proceedings. I count one MBA, two AOA staff members, one Resident and 10 accomplished physicians with enough years of medical experience to match my age many times over, never mind the 20 other professionals seated across the room. Conversely, I am a 23-year-old OMS-II at Western University of Health Sciences with a B.S. from the University of Maryland and a repro exam in about a week. Amused, I shake my head and set upon unpacking my Macbook and intending to simultaneously take notes on our briefings and catch up on the lectures I missed during yesterday’s flight from LAX. I’m about six hours behind in class, but this will be such a unique opportunity for both learning and networking, that I would be ill-advised to squander it in favor of studying. A short scan of the meeting handbook reveals an unsurprisingly ambitious agenda for the two-day session. With presentations on disaster preparedness and rural health disparities, updates on advocacy and strategic goals and a report from the AOA department of government affairs, the day is topped off by six consecutive 30-minute appointments with congressional aides to notable politicians. Tomorrow looks a bit lighter, with only four different sessions crammed into about a fourth of the time we have today.
page 34
As the meeting begins, each Bureau member introduces themselves and provides an update on the projects and practices they have managed since the last meeting. I’m pretty sure no one wants to hear about ovarian cysts or the stages of labor, so I scramble to draft something presentable. While I struggle with a seemingly spontaneous case of acute agraphia, a doctor from Ohio discusses Patient Centered Medical Homes – a concept I had not heard of before attending my first meeting last September. The next Bureau member, the Dean of a NYC medical school, explains issues with Caribbean schools that have been purchasing residency slots. A few other items are brought up by the next two members, ranging from e-cigarettes to concerns with the new physician payment model. But, before we can proceed any further, the Chair announces that two guest speakers from the U.S. Department of Health and Human Services have arrived for discussion. For the next 30
minutes the HHS Deputy Secretary and the Director of Provider Outreach proceed to field questions from the Bureau about SGR, the ACA, and other health policy acronyms. Unlike the usual lecture or seminar on these issues, their answers don’t reflect personal opinions or even expert interpretations. Rather, like most of the information presented at the Bureau meetings, everything they offer is a definitive, up-to-the-minute assessment directly from the source.
Spring 2014
The Fast Track
It truly humbles me when I realize how exclusive this opportunity is for a young medical student with no real world experience. By applying for the BFHP Student Representative position I hoped to expand my involvement in health policy. Even with my passion for the interface of politics and healthcare, I had little expectation for receiving the nomination, let alone find myself with a front row seat to this uniquely illuminating forum. Here, I could garner the firsthand perspectives of actual physicians, and obtain a true awareness on matters for which my grasp was faulty. The recent issue of Unified Accreditation embodies how such a lack of understanding distorts the student view, amplifying the distress and exasperation that often accompanies closed-door negotiations. Exclusion from the bargaining room imparts the illusory appearance that talks are stagnant and obscures the significant progress that they actually achieve. This leads students to hastily label the leadership apathetic, even when a breakthrough nears fruition. In the same way, it is very easy for a student to find reason to support the Affordable Care Act’s expansion of insurance, but much more difficult to recognize the flaws related to the revised physician payment scheme and quality reporting system. The BFHP illuminates this world of medicine and health policy from a perspective that only a practicing physician could understand, and I now see these matters with a sharper clarity and heightened awareness of their implications for the practice of medicine. As the HHS briefing draws to a close, the Chairman calls for us to resume the original order of business – introductions. I snap out of my ruminations and rewrite the nonsensical jumble sitting before me on my screen. By the time I must speak, I recite a respectable list of items that tend to worry a majority of concerned medical students (i.e. Unified Accreditation, GME funding, etc.). I reiterate my desire to learn and use the experience as a means of enriching both myself and my fellow classmates, and hurriedly take my seat. At the moment, I honestly don’t know what I could really hope to contribute to the conversations we will engage in throughout the day. I am a literal rookie among some of the profession’s all-time greatest doctors. I’m more than a bit skeptical that my input holds any credibility in this arena. And yet, this shortcoming is exactly why I’m here – to learn, not only for my own understanding, but for the benefit of my classmates and fellow students. While I am here I will absolutely speak for the students on the issues that concern us. When the topic is beyond our scope or my knowledge, I take notes. These I will craft into my own presentation for debriefing my classmates at Western U. Any knowledge or skill I receive from the Bureau will be shared. This is the aspect of Bureau service that I find most rewarding – the opportunity to impart the wealth of insight garnered from the Bureau upon my peers, so that they, too, may share a glimpse of the physician’s perspective and recognize how health policy actually plays out in the clinical setting. In five, 10, or even 20 years, I know that my brief term with the AOA Bureau on Federal Health Programs will not serve as just a notation on my résumé, but instead as a key turning point in my professional development. For any student with even an inkling of interest in leadership and health policy I urge them to apply to an AOA bureau, council, or committee. They certainly won’t regret it. References: https://www.osteopathic.org/inside-aoa/Education/students/SiteAssets/Pages/default/2014%20 Leadership%20Opportunities%20Packet.pdf
page 35
The Fast Track
Spring 2014
Case Study:
Why can’t I pass the tube?
A case of suspected ARB-induced angioedema By Drew Kalnow, DO @ Doctors Hospital History of Present Illness: A 61-year-old, African-American male presented to the ED via EMS after being found unresponsive at home by his wife. EMS reported finding the patient lying on the floor in a bedroom severely hypoxic, with agonal respirations and a strong radial pulse. EMS established peripheral IV access and performed BVM ventilations en route to the hospital with no reported difficulty.
shallow, difficult to ventilate with BVM Abdomen: Obese and distended Musculoskeletal: No edema noted Skin: Warm, dry with no obvious rashes Initial Workup and Treatment: EKG: NSR Labs: WBC 16.2, Hb 15.0, Na+ 137, K+ 5.3, Cl- 98, HCO3 30, BUN 13, Cr 1.2, Lactate 3.4, proBNP 496, CPK 246.
Past Medical History: Hypertension, diabeties Patient Course: mellitus type II, hyperlipidemia (recently Patient presents to the ED hypoxic with established care with a PCP) agonal respirations and is ventilated easily by BVM. Endotracheal intubation was attempted Current Medications: but was unsuccessful with both traditional • Amlodipine 10mg laryngoscopic and glidescopic approaches • Lipitor 40mg due to inability to pass the ET tube through • Valsartan 320mg the vocal cords. During this time, the patient • Plavix 75mg became progressively more difficult to • Potassium Chloride ventilate. Anesthesia and Surgery were notified of the difficult airway and the patient Allergies: No known drug allergies was prepared for a surgical airway. A bedside Social History: Non-smoker and occasional cricothyrotomy was performed and the patient EtOH was taken by surgery to the OR in attempt to Review of Systems: Unobtainable due to obtain more secure airway. Further airway patient condition intervention was aborted due to concerns of rapid deterioration and the patient was Physical Exam: taken to the ICU. During this period of time, BP- 195/114, P – 88, RR – 5, SpO2- 92% via BVM the patient was hypoxic with an SpO2 in the w/O2 50-70’s. Immediately upon arrival in the ICU, the patient went into cardiopulmonary arrest General : Unresponsive with agonal respirations and was successfully resuscitated twice using Neuro : Unresponsive to all stimuli ACLS. Head/Neck : Atraumatic and normocephalic Eyes: Pupils equal and reactive to light After the patient was stabilized in the ICU, a Heart: Regular rate and rhythm. No clicks, rubs, CT scan was obtained [seen at right] revealing gallops or murmurs severe supraglottic and glottic airway swelling Lungs: Breath sounds clear and equal but via CT. Otorhinolaryngology was consulted page 36
and suspected the swelling was secondary to angioedema or epiglottitis. The patient was treated with steroids and an H2 blocker, resulting in significant improvement. Ultimately, the patient was extubated and discharged from the hospital in stable condition.
suspected to directly affect kinin metabolism, recent animal studies have suggested that ARB’s may increase bradykinin production. Regardless of the cause, due to the short period of cardiac arrest and subsequent postarrest hypotension, this patient was dosed IV 1:10,000 Epinephrine and started on an Discussion: Epinephrine drip for the first few hours after A patient presenting in respiratory failure ROSC. The epinephrine, combined with the with the discovery of a difficult airway, while addition of Decadron following the evidence attempting to obtain a secure airway, is one of of significant airway edema via CT and the most daunting events that can occur in the laryngoscopy, likely served as appropriate and ED. This case presented many challenges to definitive treatment. the receiving EM team, the surgery team and ultimately the ICU team due to both the acute nature of the patient’s airway compromise References: and subsequent hemodynamic deterioration. 1. Bauman K. Endotracheal tube management During the initial intubation, it was discovered and complications. UptoDate website. http:// that the airway was so edematous, passing www.uptodate.com. Accessed Jan 23, 2014. an ET tube through the vocal cords was 2. Bingham C. An overview of angioedema: impossible. That same edema posed significant Clinical features, diagnosis, and management. challenges in securing a surgical airway due to UptoDate website. http://www.uptodate. a loss of landmarks, significant bleeding and com. Accessed Jan 23, 2014. difficulty in securing the airway device. 3. Bingham C. An overview of angioedema: Pathogenesis and causes. UptoDate website. ARB-induced angioedema was the suspected http://www.uptodate.com. Accessed Jan 25, cause of the acute airway edema and 2014. 4. Wilkerson RG. Angioedema In The subsequent respiratory failure in this patient. Emergency Department: An Evidence-Based Though ARB-induced angioedema is rare and Review. Emergency Medicine Practice. 2012; significantly less likely than with the use of ACE 14(11). http://www.EBMedicine.net. Accessed inhibitors, the ONTARGET trail found a 0.1% Jan 23, 2014. incidence (0.3% for ACEi). While ARB’s are not
Spring 2014
The Fast Track
page 37
The Fast Track
APRIL
Spring 2014
24
FOOD, FUN & FRIENDSHIP Tanzy Restaurant, Scottsdale, AZ 7pm â&#x20AC;&#x201C; 10pm Join the ACOEP and TEAMHealth at Tanzy for a night of food, fun, and friendship and to learn about the GREAT opportunities TEAMHealth has to offer for you and your career in Emergency Medicine. To learn more about what TEAMHealth have to offer visit www.teamhealth.com page 38
Circling the Wagons
Spring 2014
The Fast Track
by Andy Little, DO @ Doctors Hospital
D
eath, tragedy; we’ve all been there. Those of us who work in the emergency department have been witnesses to calamities that leave you with a pit in your stomach. We have been exposed to things that would leave most humans shattered. But, because we do what we do, we have to find ways to shrug that experience – those feelings – off and go on to the next patient who deserves our undivided attention. That being said, every once in a while there comes a case that you hope happens at the end of your shift, or when things aren’t too busy. The case where you get a gut check, have to hold back your anger or any other emotions you might be feeling, and you pray that you get some time to let this one go. It could be a child that was badly abused, the tragic death of a young person, a co-worker’s family member, a staff member who collapsed and died while at work. The list could go on. And despite our training that teaches us to disconnect and go onto the next, sometimes we need more than that, we need to stop and think. We need to feel. When I think of times like these I think of pioneers who crossed divides (the plains, the Rocky Mountains, etc.). The Oregon Trail kind of people, who after long days filled with pitfalls, and sometimes tragedy, would circle the wagons. Now Hollywood would make you believe this was done solely as a way to protect against Native Americans, bandits, and other sorts of danger. But I’m a firm believer circling the wagons was done more as an opportunity for these groups to rally behind each other, to discuss the day, learn from each other’s experiences and to grow closer through adversity. Sometimes this would be done over a meal, would include song, dance and laughter. And for a moment they forgot about the long day, the loss of a family member or the miles still to go. What can we in the field of emergency medicine do to circle the wagons? I think this is personalized to each individual. Some good general rules are to go out and spend time with your co-workers away from work. Whether it be going to a sporting event, a movie or just out to eat, this allows you to find common ground with your co-workers outside of the department and even gives you an opportunity to vent a little. One thing we as a whole don’t do enough of is post-scenario “wrap ups;” these are a great way to take a few minutes with everyone who was in the room or involved to discuss what went well, what we can improve on, and an opportunity for some emotions to be shared. I’ve found these to not only be helpful as a teaching opportunity, but as a way to move on for the rest of the day. Before I started doing these, I would find myself carrying emotions and feelings around for the rest of my shift. So as each of us continue to move forward working in our respective emergency departments, I hope we each take the time to “circle the wagons.” Because, as with the nature of our field, it isn’t if you have a tragic case, it’s when, and I hope each of us can be ready to deal with the appropriate emotions and feelings when the time comes. +
page 39
The Fast Track
Spring 2014
Residency Spotlight Midwestern University – Chicago, Illinois –
Size: 16 residents per year, 68 total ED volume, hospital sizes: We rotate at up to ten different hospitals around the city with individual volumes averaging from 30-136k/year. But more importantly, you are usually the only resident on and often the only resident in the whole hospital, so you get your pick of patients and all procedures are yours! Our hospitals range from inner city to rural populations, allowing you the most comprehensive education you can find.
page 40
We also have the incredible luxury of training at the famous Stroger Hospital of Cook County for both our trauma and toxicology rotations. Having this on your résumé is an item that will be highly sought after.
The Fast Track
residency including AOA, ACOEP, and ACEP national leadership, international With just under 70 current residents, 32 health, tactical medicine fellowships, years of experience, and more than 450 disaster medicine, medical school education alumni, our program is the largest AOA and mentorship… to name a few. emergency medicine residency, and one of the oldest, most established programs We also benefit from our affiliation with M i d w e s t e r n’s in the country. Not only do we rotate medical school, through numerous which gives hospitals allowing us to “THE MOST us access to learn various EMR’s and IVE NS HE RE MP CO great teaching hospital systems but we also have one of the most EDUCATION YOU CAN FIND” opportunities, libraries, labs, vast alumni networks out faculty and there, which helps ensure educational resources we may you will find the career of not have had at a stand-alone, hospitala lifetime after residency. based residency. As an ER doctor, one goal should be that you are able to practice in any environment Describe the program in three words: and to be ready for any situation that might Aged to perfection. present itself; our program prepares you for that. It ensures you learn the adaptability you will need to succeed in your career.
Spring 2014
What is unique about your program?
What is there to do outside of work? This is the city of Chicago… that says enough! Whether you like the arts, music, professional sports, culture, cuisine, lakes, golf, biking, shopping or anything else... you can find it in this city. We have residents that live right downtown on Lake Michigan and others that live out in the country of Indiana – so no matter which you prefer, you will find a place to live that fits you perfectly. The residents in our program also thrive on various professional opportunities outside of just
page 41
Review Answers
The Fast Track
Spring 2014
Question 1: Answer B. Optic neuritis results in monocular vision loss from demyelination of the optic nerve. Most commonly, patients are between the ages of 15 and 45 years and there is a 30-40% association with multiple sclerosis. Examination reveals decreased visual acuity and an afferent pupillary defect. Fundoscopic examination demonstrates a normal or swollen optic disk. Without treatment, patients progress to their poorest vision in approximately one week with slow improvement over the next several weeks. Steroid therapy may be started however its long-term benefit remains unclear. Macular degeneration (A) characteristically causes progressive loss of central vision. The most common form is age-related macular degeneration although it may result from trauma, radiation exposure, inflammatory or infectious disease, vascular disease or hereditary disease. Visual loss continues and it is the leading cause of legal blindness in the US. Papilledema (C) may appear similar to optic neuritis on fundoscopic examination although more commonly is bilateral. In papilledema, swelling of the optic disk results from increased intracranial pressure. The pupil examination and visual acuity should remain normal and ocular pain is typically absent. Retinal detachment (D) may cause visual loss classically beginning with flashes of light or floaters followed by a falling curtain over their visual field. Bedside ultrasound can demonstrate the separation of retinal layers from accumulation of fluid aiding in the diagnosis. Ref: Sharma R, Brunette DD: Headache, in Marx JA, Hockberger RS, Walls RM, et al (eds): Rosen’s Emergency Medicine: Concepts and Clinical Practice, ed 8. St. Louis, Mosby, Inc., 2013, (Ch) 71: pp 922-926.
Question 2: Answer: D. Immersion foot is a common cold injury caused by repeated exposure to wet cold temperatures above the freezing point. It progresses slowly over days resulting in neurovascular damage but not the formation of crystals. Commonly patients wear wet or sweaty socks that are not removed when wet. Patients develop cool, pale feet and complain of paresthesias. With time, the feet become cyanotic, cold and edematous. After rewarming, the skin becomes erythematous and is painful to palpation. Neurovascular problems are typically reversible. Chilblain, or pernio, (A) results from repetitive exposure to dry cold. This is a mild form of injury resulting in cold sores classically on the face, dorsa of the hands and feet and pretibial area of the leg. The sore may begin as pruritic, mildly erythematous lesions and progress to plaques, blue nodules or ulcerations. Patients with a history of Raynaud’s, lupus or antiphospholipid antibody syndrome are at higher risk. Frostbite occurs when tissues are exposed to temperatures below 0°C. Ice crystals form causing water to shift out of cells into the extracellular space leading to cellular dehydration. Microvascular injury also occurs with tissue ischemia and necrosis. All patients have some sensory abnormality. Patients also complain of clumsiness and a “chunk of wood” sensation in the affected extremity. Tissue appears mottled, waxy, violaceous white or pale yellow. Rapid rewarming is indicated. Frostnip (C) is not a true frostbite, but rather a superficial injury characterized by temporary numbness and tingling which resolves after rewarming. Neither deep tissue destruction nor crystal formation occurs. page Ref: Zafren K, Danzl DF: Frostbite, in Marx JA, Hockberger RS, Walls RM, 42 et al (eds): Rosen’s Emergency Medicine: Concepts and Clinical Practice, ed 8. St. Louis, Mosby, Inc., 2013, (Ch) 139: pp 1877-1884.
Question 3: Answer: A. Radiation exposures affect multiple organ systems in the body, particularly those undergoing rapid cell turnover (e.g. hematopoietic and gastrointestinal systems). Early development of gastrointestinal symptoms is associated with higher exposures and worse prognosis. Objectively, the absolute lymphocyte count is the best prognosticator at 48 hours after exposure. Given the high cell turnover of the white blood cells, they are particularly sensitive to the radiation exposure. The value of the lymphocytes is correlated both with the amount of total exposure and overall survival prognosis. An absolute lymphocyte count less than 1500 cells/mm3 is associated with a poorer prognosis.
Spring 2014
The Fast Track
A CT scan of the brain (B) is not a helpful test in evaluating a radiation exposure patient. Although CNS cells have lower rates of cell turnover, symptoms may develop as part of the CNS syndrome including: ataxia, altered mental status and seizures. Neuroimaging with CT will not help identify these patients radiographically. A Geiger counter (C) may be used in the assessment of patients for possible exposure identifying the presence of radioactive substances and help guide decontamination. However, it does not assist in prognostication. All cell lines of the bone marrow are affected after a radiation exposure including the platelet count (D) as part of the hematopoietic syndrome. The lymphocyte count is most useful in assessing degree of exposure and prognosis more than red blood cells or platelets. As part of the syndrome, patients may develop fever, infection, petechiae and hemorrhage. Ref: Colwell CB: Radiation injuries, in Marx JA, Hockberger RS, Walls RM, et al (eds): Rosenâ&#x20AC;&#x2122;s Emergency Medicine: Concepts and Clinical Practice, ed 8. St. Louis, Mosby, Inc., 2013, (Ch) 146: pp 1945-1951.
Question 4: Answer: A. When approaching the poisoned patient, the constellation of symptoms often fits into a toxidrome. This patient has characteristic features of an anticholinergic toxidrome caused by the anticholinergic properties of diphenhydramine. Central nervous system poisoning causes acute delirium and mild temperature elevation. The loss of cholinergic inhibition on the heart leads to tachycardia. Inhibition of secretory functions causes dry mouth and skin. Pupillary dilation occurs as a result of loss of cholinergic constriction of the ciliary muscles. Phenylephrine (B), a common ingredient in decongestants, will lead to a sympathomimetic toxidrome when taken in large quantities. Patients exhibit findings consistent with activation of the sympathetic nervous system: tachycardia, hypertension, dilated pupils, piloerection and diaphoresis. In severe cases, an excited delirium may occur. Salicylate (C) toxicity may lead to an elevated body temperature due to uncoupling of the oxidative phosphorylation pathway. Initially, patients have a respiratory alkalosis from activation of the respiratory center by salicylate and then develop an additional metabolic acidosis from the elevated salicylate levels. Sertraline (D) is a selective serotonin reuptake inhibitor (SSRI). The serotonin syndrome is characterized by agitation, altered mental status, fever, myoclonus, hyperreflexia, ataxia, diaphoresis and sometimes diarrhea. The syndrome may occur from a large ingestion of an SSRI but also may occur with certain drug interactions of agents that have reuptake inhibitory properties. Ref: Kulig K: General Approach to the Poisoned Patient, in Marx JA, Hockberger RS, Walls RM, et al (eds): Rosenâ&#x20AC;&#x2122;s Emergency Medicine: Concepts and Clinical Practice, ed 8. St. Louis, Mosby, Inc., 2013, (Ch) 147: pp 1954-1956. page 43
The Fast Track Keep an eye out for our next issue! Spring 2014
The Fast Track
Summer Edition Preview!
LICATION AN EMERGENCY MEDICINE PUB
CGME Merger ACOEPâ&#x20AC;&#x2122;s Response to the AOA-A
Wilderness Medicine
The Future of Backboards
Spring Conference Recap The return of the Ultrasound Corner
ACOEP Resident and Student Chapter 142 East Ontario Street Suite 1500 Chicago, Illinois 60611 Phone: 312.587.3709 page Fax: 312.587.9951 44 E-mail: fasttrack@acoep.org
Follow us on
@ ACOEPSC @ ACOEPRC @ facebook.com/acoepsc @ facebook.com/acoeprc